Finden Sie orthogonale Vektoren in 4 Dimensionen

Ich habe 4 Vektoren ein R 4 wie die folgenden:

A 1 , A 2 R 4  Und  A 1 A 2 , B 1 , B 2 R 4  Und  B 1 B 2 .
Ich habe sie so konstruiert, dass ich zufällige Vektoren aufgenommen habe R 4 und lief Gram-Schmidt auf Paaren ( A 1 , A 2 ) Und ( B 1 , B 2 ) . Jetzt möchte ich eine Situation, in der
A 1 B 1 , A 2 B 2 .
Ich kann Gram-Schmidt wieder auf die Paare anwenden ( A 1 , B 1 ) Und ( A 2 , B 2 ) , aber dann verliere ich die Eigenschaft, dass ( A 1 A 2 ) Und ( B 1 B 2 ) . Gibt es eine Möglichkeit, die oben genannten Wünsche zu erfüllen? Beachten Sie, dass ich Gram-Schmidt ausführen könnte ( A 1 , A 2 , B 1 , B 2 ) finden 4 orthogonale Vektoren, aber das will ich nicht. Beachten Sie, dass ich in meiner Spezifikation nichts über die inneren Produkte dazwischen sage A 1 , B 2 oder A 2 , B 1 . Gibt es eine Möglichkeit, dies zu tun? Oder kann ich beweisen, dass es unmöglich ist? Danke!

Um es klar zu sagen, Sie möchten solche Vektoren generieren A 1 B 1 B 2 A 2 A 1 (ohne weitere Bedingungen für die restlichen inneren Produkte)? (Ich wäre auch neugierig, in welchem ​​Kontext Sie darauf stoßen. Es erinnert mich an Berechnungen, die ich bezüglich der sogenannten CHSH-Ungleichung in der Quantenmechanik gesehen habe.)
Sie haben Recht, das ist die Bedingung, die ich suche. Ich habe in einem Kryptografie-Szenario nach diesen Vektoren gesucht. Ich weiß nicht, wie diese mit dem CHSH-Spiel zusammenhängen könnten. Dein Punkt ist sehr interessant!
Ich weiß nicht, ob dies Ihre Frage beantworten würde, aber gegeben N 1 Vektoren A 1 , , A N 1 R N Sie können immer einen neuen Vektor finden A N orthogonal zu A J für J = 1 , , N 1 . Dies kann durch Verallgemeinerung des Kreuzprodukts in bewiesen werden R N und Definition der ich -te Koordinate von A N als ( 1 ) ich det A ich , Wo A ich ist der ( N 1 ) × ( N 1 ) Matrix erhalten durch Löschen der ich -te Reihe der N × ( N 1 ) Matrix A := ( A 1 , , A N 1 ) .
Ein Ansatzpunkt, um zu zeigen, dass nicht-orthogonale Versionen möglich sind: Nehmen Sie einfach B 2 = A 1 Und B 1 = A 2 . Dann die Orthogonalität von A 1 Und A 2 sorgt dafür, dass jedes Paar außer A 1 , B 2 Und A 2 , B 1 sind orthogonal (weil sie natürlich parallel sind).
I. Roperval, in der Tat, das gibt mir einen Vektor orthogonal zu allen anderen. Was ich möchte, ist eine schwächere Bedingung, bei der wir zulassen, dass einige Paare nicht orthogonal sind. Halbklassisch, ich verstehe, du hast recht. Es ist also möglich...
Um der Quantenverbindung ein wenig Kontext hinzuzufügen: Die Korrelationen zwischen den beiden Paaren von Observablen von Alice und Bob im CHSH-Aufbau entsprechen direkt den inneren Produkten zwischen Vektoren A 1 , A 2 , B 1 , B 2 R 3 . Eine naheliegende Frage ist, welche Quartette von inneren Produkten ( A 1 B 1 , A 1 B 2 , A 2 B 1 , A 2 B 2 ) können so generiert werden. Aber in dieser Umgebung kümmert man sich normalerweise nicht um die verbleibenden zwei Produkte, sodass die "parallele" Konstruktion gut funktioniert (und insbesondere in 3D funktioniert).
Für weitere mathematische Details ist das nachzuschlagende Wort wahrscheinlich "Elliptop". Ich kenne keine große Quelle dafür, aber das Buch von Deza und Laurent spricht ausführlich darüber.
Hübsch! Ich würde die CHSH-Verbindung und das Elliptop-Objekt überprüfen. Vielen Dank @Semiclassical .

Antworten (2)

Hier ist ein einfaches Beispiel für solche Vektoren, das es dennoch schafft, alle möglichen Winkel zwischen den unbeschränkten Paaren darzustellen:

A 1 = e 1 , A 2 = e 3 cos a + e 4 Sünde a , B 1 = e 3 , B 2 = e 1 cos β + e 2 Sünde β ,

Dann tatsächlich A 1 B 1 B 2 A 2 A 1 , aber der Winkel dazwischen A 2 , B 1 Ist a und der Winkel dazwischen A 1 , B 2 Ist β . (Dies ist natürlich ein sehr einfaches Beispiel, aber wir können eine beliebige Drehung anwenden, um ein „zufälliges“ Quartett mit solchen Beziehungen zu erstellen.)

Endlich habe ich sowohl deine als auch @lonzas Antwort verstanden. Ich akzeptiere deine, weil sie früher veröffentlicht wurde. Vielen Dank, Jungs!
@leftarondabout Ahh, das ist wirklich einfach. Wir beginnen also mit der standardmäßigen orthonormalen Basis und wenden zwei unabhängige Givens-Rotationen an, um die gewünschten Winkel zu erreichen.

Wenn   X 1 , X 2 , X 3 , X 4   sind "zufällige" unabhängige Vektoren in   R 4   , Und

j 1 = X 1 j 2 = X 2 X 2 , j 1 j 1 2 j 1 j 3 = X 3 X 3 , j 2 j 2 2 j 2 z 3 = j 3 j 3 , j 1 j 1 2 j 1 j 4 = X 4 X 4 , z 3 z 3 2 z 3 X 4 , j 1 j 1 2 j 1   ,
Dann   j 1 j 2 j 3 j 4 j 1   . Ist es das, was Sie suchen?

Beim Testen in Mathematica scheint dies in Bezug auf innere Produkte zu überprüfen. Es gibt jedoch eine seltsame Funktion. Lassen θ J k sei der Winkel zwischen den Vektoren j J , j k . Wie erwartet, θ 12 = θ 23 = θ 34 = θ 14 = 0 innerhalb der numerischen Genauigkeit. Aber es scheint zahlenmäßig so, als ob θ 24 gleichmäßig verteilt ist ( 0 , π ) , während cos θ 13 gleichmäßig verteilt ist ( 1 , 1 ) . Das ist also eine interessante Asymmetrie, vermutlich aufgrund der schrittweisen Natur dieses Gram-Schmidt-Prozesses.
Interessant. Wie haben Sie die Auswahl zufällig getroffen?   X ich S. Unabhängige Uniform über der Einheitskugel?
Nein, ich habe die Komponenten als standardmäßige normale Zufallsvariablen ausgewählt. Aber es ist einfach genug für mich, diese Vektoren zu normalisieren, bevor ich fortfahre, also lass mich das testen. Bearbeiten: Vorläufige Ergebnisse deuten darauf hin, dass die erste Normalisierung die Verteilung nicht ändert. Ich nehme an, der Punkt ist, dass die Neuskalierung in die Berechnung von absorbiert wird j k und so beeinflusst nichts.
Gute Antwort @lonzaleggiera , könnten Sie mir bitte beim nächsten Fall helfen? Wenn ich 9 neundimensionale Vektoren a1, a2, a3.. b1, b2, b3.. c1, c2, c3.. habe, wobei diese Tripel orthogonal zueinander sind und a1 \perp b1 \perp c1.... a2 \perp b2 \perp c2.... a3\perp b3 \perp c3. Ist diese Konstruktion prinzipiell möglich? Mir macht es nichts aus, mich anzustrengen...
Ich habe mich im vorherigen Kommentar ein wenig vertan, zusammen mit den Elementen der Triolen (a1, a2, a3). (b1, b2, b3) und (c1, c2, c3) sind orthogonal zueinander (zum Beispiel: a1 \perp a2, a3 \perp a1 und b1 \perp b3 usw.), möchte ich auch, dass (a1, b1, c1), (a2, b2, c2) und (a3, b3, c3) orthogonal zueinander sind .. Ist es möglich?
Angesichts der Tatsache, dass Sie ein bestimmtes Problem im Sinn zu haben scheinen, könnte eine andere Frage angebracht sein … @QuestionEverything
Okay @Semiclassical , das könnte besser sein. Danke!
Hier ist die neue Frage gemäß Ihrem Vorschlag @Semiclassical: math.stackexchange.com/questions/4224461/…